Skip to content

Instantly share code, notes, and snippets.

@brando90
Created May 9, 2024 21:31
Show Gist options
  • Save brando90/e48dc29d2ab80e82998d0f78fd9bf4e1 to your computer and use it in GitHub Desktop.
Save brando90/e48dc29d2ab80e82998d0f78fd9bf4e1 to your computer and use it in GitHub Desktop.
minerva_prompt_cot.py
# -- Prompt minerva MATH 3 - better minerva + cot/scratch_pad
# https://github.com/EleutherAI/lm-evaluation-harness/blob/main/lm_eval/tasks/minerva_math/utils.py#L22
H_MATH_MINERVA_PROMPT_TEMPLATE_3_COT = (
r"""Problem:
Find the domain of the expression $\frac{\sqrt{x-2}}{\sqrt{5-x}}$.}
Solution:
Let's think step by step. The expressions inside each square root must be non-negative. Therefore, $x-2 \ge 0$, so $x\ge2$, and $5 - x \ge 0$, so $x \le 5$. Also, the denominator cannot be equal to zero, so $5-x>0$, which gives $x<5$. Therefore, the domain of the expression is $\boxed{[2,5)}$.
Problem:
If $\det \mathbf{A} = 2$ and $\det \mathbf{B} = 12,$ then find $\det (\mathbf{A} \mathbf{B}).$
Solution:
Let's think step by step. We have that $\det (\mathbf{A} \mathbf{B}) = (\det \mathbf{A})(\det \mathbf{B}) = (2)(12) = \boxed{24}.$
Problem:
Terrell usually lifts two 20-pound weights 12 times. If he uses two 15-pound weights instead, how many times must Terrell lift them in order to lift the same total weight?
Solution:
Let's think step by step. If Terrell lifts two 20-pound weights 12 times, he lifts a total of $2\cdot 12\cdot20=480$ pounds of weight. If he lifts two 15-pound weights instead for $n$ times, he will lift a total of $2\cdot15\cdot n=30n$ pounds of weight. Equating this to 480 pounds, we can solve for $n$:
\begin{align*}
30n&=480\\
\Rightarrow\qquad n&=480/30=\boxed{16}
\end{align*}
Problem:
If the system of equations
\begin{align*}
6x-4y&=a,\\
6y-9x &=b.
\end{align*}has a solution $(x, y)$ where $x$ and $y$ are both nonzero,
find $\frac{a}{b},$ assuming $b$ is nonzero.
Solution:
Let's think step by step. If we multiply the first equation by $-\frac{3}{2}$, we obtain
$$6y-9x=-\frac{3}{2}a.$$Since we also know that $6y-9x=b$, we have
$$-\frac{3}{2}a=b\Rightarrow\frac{a}{b}=\boxed{-\frac{2}{3}}.$$
Problem:
{problem}
Solution:
Let's think step by step."""
)
Sign up for free to join this conversation on GitHub. Already have an account? Sign in to comment